how do you know that the expression 2x + 4y is not equivalent to 6xy

Answers

Answer 1

Answer:

The expression is not equivalent because you cannot combine 2 variables that are different from eachother.

Step-by-step explanation:

you are able to add 2+4 but because they have variables with them, x and y, you can not add them together.


Related Questions

Select the type of equations.
equivalent
inconsistent
consistent

Answers

The answer is consistent

Answer:

the answer is inconsistent hope this help :)

Step-by-step explanation:

how do I find b of the traingal​

Answers

Answer:

Pythagorean Theorum:

16ft

Step-by-step explanation:

pythagorean theorum states a^2+b^2=c^2

it only works for right triangles, c is the hypotenuse, a and b are the legs

so to find b we plug in the info we have!

7.8^2+b^2=17.8^2

next we simplify:

60.84+b^2= 316.84

subtract 60.84 from both sides to isolate the variable:

b^2=256

square root both sides

b=16

16ft is the answer

I hope this helps!

1. Isosceles Triangle The two equal sides of an isosceles triangle are each 42

centimeters. If the base measures 30 centimeters, find the height and the

measure of the two equal angles. Round your answer to the nearest tenth.

Answers

Answer:

1. H = 29 cm

2. θ = 44°

Step-by-step explanation:

1. We can find the height of the triangle by considering the isosceles triangle as two right triangles. The height can be found by using Pitagoras:

[tex] L^{2} = H^{2} + B^{2} [/tex]

Where:

L: is the sides of the isosceles triangle = 42 cm

B: is the base = 30 cm

H: is the height =?

Then, the height is:

[tex] H = \sqrt{L^{2} - B^{2}} = \sqrt{(42 cm)^{2} - (30 cm)^{2}} = 29.4 cm = 29 cm [/tex]

2. The two equal angles (θ) can be found using the following trigonometric identity:          

[tex] cos(\theta) = \frac{B}{L} [/tex]

[tex] \theta = cos^{-1}(\frac{30 cm}{42 cm}) = 44.4^{\circ} = 44^{\circ} [/tex]

Hence, the two equal angles are 44°.

I hope it helps you!  

A local yoga studio
charges a sign-up fee of
$55, plus $10 per class. A
second yoga studio
charges a sign-up fee of
$25 plus $12.50 per class.

A. Griffin
B. Gigi

Answers

Answer:

Let's call the first studio, yoga studio A.

Let's call the second studio, yoga studio B.

The equations:

Yoga Studio A: y=10x+55

Yoga Studio B: y=12.5x+25

So, for 12 classes:

Yoga Studio A: y=10(12)+55, y=175

Yoga Studio B: y=12.5(12)+25, y=175

These two numbers are equal, so Griffin is right.

For 10 classes:

Yoga Studio A: y=10(10)+55, y=155

Yoga Studio B: y=12.5(10)+25, y=150.

These two numbers are not equal, so Gigi is wrong.

Let me know if this helps!

8 increased 3 times by a number t​

Answers

Answer:

3t+8

Step-by-step explanation:

Answer:

3t + 8

Step-by-step explanation:

Let the number be represented by

t

Now Interpreting the statement;

Three times a number;

3 × t

3t

Increased by eight;

3t+8

I hope this is helpful

please mark it as brainlist

A, C, E
A, C, D
B, D, E
C, D, E

Answers

Answer: A C and E (1st choice)

Step-by-step explanation:

(33x3)+(33x3)= blah blah blah sorry it had to be 20 characters long ;)

Answers

The answer is 135...............

Answer:

198

Step-by-step explanation:

due to order of eliminations u multiply 33 and 3 in both parantheses. then you add them together, you get 198.

please help me with this !!!

Answers

Answer: Hope this helps
No it is -2 hope this helps

What is the y-intercept of the line with the equation 4y = -3x + 10?

Answers

Step-by-step explanation:

Given

4y = - 3x + 10

y = ( - 3x + 10 ) / 4

y = - 3/4 x + 10 / 4

y = - 3/4 x + 5 / 2

Comparing the given equation with y = mx +c Then

y - intercept (c) = 5/2

Hope it will help :)

PLEASE HELP !!!
In an 80/20 mortgage, what is the first mortgage used for

A. 80% down payment
B. 20% interest rate
C. 20% down payment
D. 80% of the home’s value

Answers

Answer: D. 80% of the home’s value

============================================================

Explanation:

As you probably expect, the first number 80 refers to the percentage the first loan covers. If the house is say $100,000, then the first loan is $80,000 while the second loan is the remaining $20,000.

An 80/20 mortgage, or similar, will have two monthly payments because you are getting two mortgages bundled together. Usually you should pay a down payment, though it may likely depend on your credit history. Those with good credit will pay less or no down payment, compared to those with worse credit will have to pay more down payment. A good rule of thumb is that 20% of the home's value is made as down payment, though this isn't what the "20" in "80/20" is referring to.

An 80% down payment is extremely high and unreasonable. Not many people have that kind of money laying around. A similar story applies to a 20% interest rate which is incredibly large for a mortgage rate (typically they are in the single digits such as 3%).

Please help!
Write down the correct type of angle for angles A to E.

Answers

Answer:

Angle A- Obtuse angle

Angle B- Right angle

Angle C- Reflex angle

Angle D- Reflex angle

Angle E- Acute angle

Angle A is Obtuse angle, Angle B is  Right angle, Angle C is Reflex angle, Angle D is Reflex angle and Angle E is Acute angle

What are Angles?

An angle is the figure formed by two rays, called the sides of the angle, sharing a common endpoint, called the vertex of the angle.

Obtuse angle is any angle greater than 90°

A right angle is an angle measuring exactly 90°

A reflex angle is an angle that is more than 180 degrees and less than 360 degrees.

Any angle that measures greater than 0° and less than 90° is called an acute angle.

By the definitions of these the angles are

Angle A- Obtuse angle

Angle B- Right angle

Angle C- Reflex angle

Angle D- Reflex angle

Angle E- Acute angle

Hence, Angle A is Obtuse angle, Angle B is  Right angle, Angle C is Reflex angle, Angle D is Reflex angle and Angle E is Acute angle

To learn more on Angles click:

https://brainly.com/question/28451077

#SPJ2

The cylinder on a tanker truck is 27 feet long and has a radius of five feet. What is the volume of the tank?

Answers

Answer:

2120.85 ft^3

Step-by-step explanation:

Given data

Height h= 27 ft

Radius r= 5 ft

The volume of a cylinder is given as

V= πr^2h

Substitue

V= 3.142*5^2*27

V= 3.142*25*27

V= 2120.85 ft^3

Hence the volume is 2120.85 ft^3

ASIGNATURA: Matemáticos
Tema: Una respuesta inesperada
Instrucciones: Lena los espacios escribiendo el número que resulta de multiplicar el número
de la columna de la izquierda por cada uno de los números de la fila superior.
3
5
4
6
2
10
8
4
3
27
21
18
5
15
45
40
7
63
49
14
16
8
72
24
18
72
36
54
45
9
8
12
28
36
a) Los múltiplos de 3 son:
b) Todos los números pares son múltiplos de:
c) Los múltiplos de 5 son:
dos en negrita​

Answers

Answer:

a) Los múltiplos de 3 son: 3,6,9,12,15,18,21,24,27

b) Todos los numeros pares son multiplos de: 2

c) Los múltiplos de 5 son: 5,10,15

how much of a radioactive kind of protactinium will be left after 28 hours if you start with 672 grams and the half-life is 7 hours?

Answers

Answer:

Step-by-step explanation:

672·0.5^(28/7) = 672·0.5⁴ = 42 grams

Express answer as an inequality: -13+ a > 2a - 20??​

Answers

Answer:

7 >a

Step-by-step explanation:

-13+a>2a-20

-13+20 >2a-a

7 > a

lm=6x+3 mn=4x-17 ln=x+14 (find the range of possible values for x, help a sister out)

Answers

Answer:

Step-by-step explanation:

lm=6x+3

mn=4x-17

ln=x+14

lm+mn>ln

6x+3+4x-17>x+14

10x-14>x+14

9x>28

x>28/9

mn+ln>lm

4x-17+x+14>6x+3

5x-3>6x+3

-x>6

x<-6

lm+ln>mn

6x+3+x+14>4x-17

7x+17>4x-17

3x>-34

x>-34/3

combining x<-6 or x>28/9

Answer and explain pleaseeeee.

Answers

Answer:

Answer : - diagonals of rhombus are perpendicular. According to the property of rhombus. Diagonals of rhombus are perpendicular. In this question the triangle NQM is a right angled triangle and it is clear that it is right angled triangle because he mentioned angle MQN = 90°

The ratio of pennies : nickels is proportional to the ratio of nickels : dimes,
and to the ratio of dimes : quarters. If you have one penny and two nickels,
how much money do you have?

Answers

Wee I don’t know I gun

What is the value of the fourth term in a geometric sequence for which ay =
10 and r=0.5?
Answer here
I
SUBMIT

Answers

Answer:

a₄ = 1.25

Step-by-step explanation:

Given that,

First term of GP, a₁ = 10

The common ration of GP, r = 0.5

The nth term of the GP is given by :

[tex]a_4=a_1r^{n-1}\\\\a_4=a_1r^3\\\\a_4=10\times (0.5)^3\\\\a_4=10\times 0.125\\\\a_4=1.25[/tex]

So, the fourth term of the GP is 1.25.

Look at the angles marked a, b, c and d.
Write the letter of the angle alongside its special name.
acute angle
reflex angle
right angle
obtuse angle

Answers

Answer:

acute angle : C

reflex angle : B

right angle : D

obtuse angle : A

The FBI wants to determine the effectiveness of their 10 Most Wanted list. To do so, they need to find out the fraction of people who appear on the list that are actually caught. Step 2 of 2 : Suppose a sample of 897 suspected criminals is drawn. Of these people, 322 were captured. Using the data, construct the 95% confidence interval for the population proportion of people who are captured after appearing on the 10 Most Wanted list. Round your answers to three decimal places.

Answers

Answer:

The answer is "0.359"

Step-by-step explanation:

Throughout the given question, in the sample of 897 suspected offenders were identified, 322 have also been convicted. So, the estimate of the population of individuals caught and been on the ten most attractive lists is then estimated:

[tex]\to \frac{322}{897} = 0.3589743.. \approx 0.359[/tex]

Please help ASAP!
Thank you

Answers

The answer has to be 16 because when you add them it goes up
Yep. Your answer would be 16

A rectangular prism has a base that is 1.5 meters by 2 meters, and the prism is 4 meters
high. What is the surface area of the prism?
A 28 m
C 12 m2
B 34 m2
D 31 m²

Answers

Answer:

The answer is C. 12 m2

Step-by-step explanation:

Hope this helps!! :) :) Please mark brainlliest.

The answer is C 12m2

A circles diameter is 12 inches what is the circles circumference using 3.14 rounded to the nearest tenth

Answers

37.7 that is the exact number hopes this helps

Answer:

[tex]37.7in.[/tex]

Step-by-step explanation:

The circumference of a circle is: [tex]C=2\pi r[/tex]

However, the question wants us to use 3.14 instead of [tex]\pi[/tex].

We are given 12 inches as the diameter of the circle.

We want the radius, which is half of the diameter.

The radius of the circle is 6 inches.

Now that we have the radius, we plug and chug.

[tex]C=2*3.14*6in.\\C=6.28*6in.\\C=37.68in.\\[/tex]

However, the question wants us to round the answer to the nearest tenth.

Therefore, the new answer is: [tex]37.7in.[/tex]

[tex]\frac{t}{7}= 13\\[/tex]

Answers

Answer:

91

Step-by-step explanation:

Step 1: Multiply both sides by 7. T/7 = 13 (t/7) = (13) x 7  t = 91

what’s the question for this?

The volume of the prism is 72 cubic feet. Find the missing height of the triangular prism, and the surface area.

Answers

Answer:

h = 12

surface area = 192

Step-by-step explanation:

Volume of this prism = 72

The base triangle has an area of B * h/2

The base = 6

The height in red = 2

Area = 1/2 * 6 * 2

Area = 6 ft^2

The volume = B * h = 72

The base = 6

6* h = 72

h = 72/6

h = 12

The surface area = 2 triangles + the rectangle face closest to us + the rectangular face whose width is 4 + rectangular face whose with is 5

The surface area = 2 * 6 + 6* 12 + 4*12 + 5 * 12

The surface area = 12 + 72 + 48 + 60

The surface area = 192 square feet.

H=12 I feel like it this

Help me out here pleaseeeeeeee

Answers

11 x 16 = 176 so C is your answer

Answer:

176

Step-by-step explanation:

11ft x 16ft = 176ft

1ft x 176ft = 176ft

A pond holds 1920 gallons of water. There is a leak and the amount of water drops 20 gallons every hour. How much water is in the pond after 2 days ?

Answers

Answer:

20 times an hour so 20 times 48 is 960 1920-960 is 960 gallons so there is 960 gallons of water left

Answer:

960 gallons

Step-by-step explanation:

20 gallons * 48 hous =960 gallons

1920 - 960 =960 gallons.

Using the graph of f, determine f(0).

Answers

when x = 0, y = 0

f(0) = 0

Calculate the slope of graph E. Type your answer in the box.

Answers

There are no numbers
The answer is y=0
If this helps then please consider marking this as brainliest
Other Questions
what do you feel about the message conveyed Which expression is equal to 6/7? Choose 1 answer: (Choice A) 6+7 (Choice B) 67 (Choice C) 67 (Choice D) 6-7 8. Where should you look to check a word's spelling? Select all that apply.a print dictionaryO an encyclopediaan online dictionaryspell check for a word processing program Prove that the median to the base of an isosceles triangle is also: the altitude to the base. Also, Prove that the median to the base of an isosceles triangle is also: the angle bisector of the angle from which it is drawn.Please provide statement and reason proof. Would really appreciate it. Best Answer will get brainliest, and please don't spam stuff just for points. Thanks, and have a great day! What is 789 divided by 3 3. What is the value of the digit 4 in these numbers?4 .b. 204C. 468a. 964e. 341d. 459400a0 1 2 3 44 5 6 7 8 9 10 11 4. You must build a model of an aquifer for a scienceproject. What material would be the best to usefor the layer that will hold water?a. an impermeable material, such as clayb. a liquid, such as oilc. a permeable material, such as graveld. a material that does not have pores Algebra Find the values of the variables for whichABCD is a parallelogram.3.DA(2x + 10)+ 20(2x 10)B Sam is planning to start a pool cleaning business from his home. He has decided on a radius of 30 miles from hishome as a region of operation. After surveying the region of operation, he has found that 43% of homes in theregion have a swimming pool. There are 36,248 homes in Sam's region of operation..Which of the following statements is the best reason to include in the market analysis component of Sam's businessplan explaining why such a high percentage is good for Sam's business?43% is 40% more than Sam needs.b. Only 57% of the home owners need to build a swimming pool43% of homes with a pool means 15,586 guaranteed customers, creating a solid customerbase.d. Since pools are not often removed from a property, 43% of homes with a pool means astrong potential customer base now, with the existing home owners, and later, with newhome owners.C. PLS HELP WITH THIS DENSITY QUESTIONS AND WHEN SHOWING STEPS PLS USE GRASSCalculate the density of aluminum if you have a piece of aluminum that has a volume of 48.9 cm.When the piece is measured it is found to have a mass of 132 g. A small amount of copper is found to have a mass of 27 g. Calculate the density of copper if the volume of the sample is 3.0 cm.The volume of a piece of iron is 65 cm. If the piece of iron has a mass of 512 g. Calculate the density of the sample of iron.A stick of cedar wood has a mass of 1.2 kg. It has a volume of 3243 cm . What is the density of cedar wood?Zinc has a density of 7.14 g/cm. If you are given 50 g of zinc calculate the volume of the sample.A block of aluminum occupies a volume of 15.0 mL and weighs 40.5 g. What is its density?Mercury metal is poured into a graduated cylinder that holds exactly 22.5 mL. The mercury used to fill the cylinder weighs 306.0 g. From this Information, calculate the density of mercury. What volume of silver metal will weigh exactly 2500.0 g. The density of silver is 10.5 g/cm Is AABC-AXYZ? If so, identify the similarity postulate or theorem thatapplies3030"4515959c.959A. Similar - SSSB. Similar - AAC. Similar - SASD. Cannot be determined Janet bought a book for $19.95, and the tax was 6.25%. including tax how much did she pay round to nearest cent Pls help (ZOM TO SEE FULL PIC) 10. CALCULATE: Find the amount of power used in each of the following examples. Show your calculations. a. You use a force of 10 N to move a box 100 m in 10 seconds. b. An athlete lifting weights does 900 J of work in 1 second. C. A truck does 30,000 J of work in 15 seconds. d. A fumiture mover uses a force of 150 N to push a large trunk 5 m across the floor in 5 seconds 1 pointA 40-acre farm yields 600 bushels of wheat. At the same rate, how muchwheat would a 75-acre farm yield? Jason reads for 90 minutes each day Which equation can be used to solve for m the number of minutes Jason reads in 3 days what is the surface area of the cylinder below? Let f(x) = x^2 + 1 and g(x) = x 3. Write an expression for f(g(x)). 4+(7-5) to the power of 4 Plss answer me fast im in test